LSAT and Law School Admissions Forum

Get expert LSAT preparation and law school admissions advice from PowerScore Test Preparation.

 Administrator
PowerScore Staff
  • PowerScore Staff
  • Posts: 8917
  • Joined: Feb 02, 2011
|
#32717
Complete Question Explanation

Justify the Conclusion—FIB, SN. The correct answer choice is (A)

Unlike most Fill-in-the-Blank questions, which are essentially Main Point questions in disguise, this one turns out to be a Justify question. The argument is structured as follows:
  • Premise (1): ..... Hermann does not grant interviews unless he can approve the article before publication.

    Premise (2): ..... The newspaper will not compromise its editorial integrity.

    Conclusion: ..... The newspaper will not interview Hermann.
The last sentence of the stimulus is the conclusion (hence the conclusion indicator “so” in the beginning of that sentence). Notice, however, that the clause in need of completion is prefaced by the premise indicator “since.” Clearly, then, we need to complete the passage by identifying an additional premise for the conclusion in the last sentence.

Do not let unusual-sounding questions throw you off: their presence in the very beginning of the section is not accidental. So, despite the unusual question design, our task here is a familiar one: we need to complete the stimulus in a way that enables the conclusion of the argument to be properly drawn. The sufficient condition indicator (“if”) in the question stem is a reminder that you must select an answer that is sufficient to prove the conclusion by using the Justify Formula:
  • Premises + Answer choice = Conclusion
As with most Justify questions, there is a logical gap between the premises and the conclusion. To identify the missing link precisely, it would be wise to examine the conditional relationships that underlie the reasoning:
  • Premise (1): ..... Interview H. :arrow: Right to approve article

    Premise (2): ..... Compromise integrity

    Conclusion: ..... Interview H.
Clearly, just because the newspaper does not want to compromise its editorial integrity does not automatically mean that Hermann will not be interviewed. To justify such a decision, we need to establish the following connection:
  • Justify Formula: ..... Right to approve article :arrow: Compromise integrity
In other words, we are looking for a statement suggesting that the newspaper’s editorial integrity would be compromised if they gave Hermann the right to approve the article before publication. This prephrase agrees with answer choice (A).

Answer choice (A): This is the correct answer choice. This choice provides the assumption which, when added to the stimulus’ premises, fully justifies the editors’ decision not to interview Hermann:
  • Premise (1): ..... Interview H. :arrow: H. approve article

    Answer choice (A): ..... Right to approve article :arrow: Compromise integrity

    Premise (2): ..... Compromise integrity

    Conclusion: ..... Compromise integrity :arrow: Right to approve article :arrow: Interview H.
Answer choice (B): This answer choice strengthens the conclusion, but it does not fully justify it. Indeed, if the newspaper has never before given an interviewee the right to approve an article before publication, then perhaps they won’t give Hermann that right either, and—consequently—not interview him. This chain of events is probable, but not certain: what has occurred in the past is no guarantee that the future will be the same. Consequently, this answer choice cannot fully justify the conclusion that the Messenger will not interview Hermann.

Answer choice (C): The willingness of most stars to grant interviews has no bearing on the question of whether Hermann would be interviewed.

Answer choice (D): The fact that Hermann usually requests substantial changes to interview articles before approving them might explain why the Messenger would be reluctant to interview him. Nevertheless, this answer choice does not definitively prove that the Messenger will not interview Hermann: our goal is to justify the conclusion, not merely support it.

Answer choice (E): Hermann’s fear of being portrayed in an unflattering light probably explains why he is reluctant to grant interviews. However, our job is not to justify a premise for the author’s argument, but rather the conclusion of that argument. And clearly, Hermann’s own rationale for not granting interviews without certain provisions cannot, by itself, explain why the Messenger will not interview him.
 lathlee
  • Posts: 652
  • Joined: Apr 01, 2016
|
#67733
Hi. Can you show how to solve this Q by using Nested conditional approach?
 Adam Tyson
PowerScore Staff
  • PowerScore Staff
  • Posts: 5153
  • Joined: Apr 14, 2011
|
#71613
Hey lathlee, there is nothing nested in this conditional relationship, so we would not take that approach. A nested conditional statement is one in which there is a conditional relationship that is itself either sufficient or necessary for another condition. For example, "if I go to the supermarket, then if they have any of your favorite beer I will buy some." The sufficient condition is "if I go to the supermarket." The necessary condition is a conditional statement: "if they have your favorite beer, then I will buy some." That type of relationship is not present in this stimulus.

In case you were wondering, the contrapositive of the above would be "if they have your favorite beer and I don't buy it, then I did not go to the supermarket."

Get the most out of your LSAT Prep Plus subscription.

Analyze and track your performance with our Testing and Analytics Package.